Вы находитесь на странице: 1из 24

Practice Questions 1

B1 C8 2 6. A patient had been on oxygen, but it was discontinued by


1. A physical therapy plan of care for a child with spastic physician order yesterday. During physical therapy, the
cerebral palsy who is three years-old chronologically and patient becomes short of breath and requests
cognitively, but at a six month-old gross developmental supplemental oxygen. The patient’s SaO2 is measured at
level would include: 90%. The decision that is NOT appropriate is:
A. reaching for a black and white object while in the A. administer supplemental O2.
supine position. B. allow the patient to rest.
B. reaching for a multicolored object while in an C. continue to monitor SaO2.
unsupported standing position. D. encourage an efficient breathing pattern.
C. reaching for a multicolored object while in an
unsupported, guarded sitting position. B7 C2 1
D. visually tracking a black and white object held nine 7. A 26 year-old presents with weakness and atrophy of the
inches from his face. biceps brachii resulting from an open fracture of the
humerus. Your examination includes needle
B2 C1 2 electromyography of the biceps. The muscle response you
2. A patient is receiving grade III mobilizations to regain would anticipate after the needle was inserted and prior to
normal midthoracic extension. After three sessions he asking the patient to contract the muscle is:
complains of localized pain that persists for greater than 24 A. electrical silence.
hours. Your treatment should: B. fibrillation potentials.
A. change to grade II mobilizations to reduce the joint C. interference patterns.
and soft tissue irritation. D. polyphasic potentials.
B. change to grade IV mobilizations to stretch through B8 C4 2
the pain. 8. You are a new staff member on the oncology unit of a large
C. change to self stretching activities because the medical center. Your first referral is for strengthening and
patient does not tolerate mobilization. ambulation for a 42 year-old women with ovarian cancer.
D. continue with grade III mobilizations followed by a She is undergoing radiation therapy following a surgical
cold pack to the thoracic spine. hysterectomy. Her current platelet count is 17,000. The
MOST appropriate treatment activity for this patient at this
B3 C7 3 time is:
3. A therapist wishes to use behavior modification techniques A. passive ROM exercise.
as part of her treatment to help shape the behavioral B. progressive stair climbing using weighted belts.
responses of a 22 year-old patient recovering from C. resistance training at 30% one repetition max.
traumatic brain injury. The BEST choice is to: D. resistance training at 50 % one repetition max.
A. allow the patient enough time for self correction of the
behavior. B9 C1 2
B. encourage the staff to tell the patient which behaviors 9. A forty-three year-old male office worker who is a long term
are correct and which are not. smoker now with emphysema has been referred to
C. reprimand the patient every time an undesirable physical therapy. He complains of increased pain and
behavior occurs. tingling in both hands after sitting at his desk for longer
D. use frequent reinforcements for all desired behaviors. than one hour. The MOST effective physical therapy
intervention is:
B4 C2 3 A. mobilization of the first rib and stretching of middle
4. Which is NOT a useful intervention for a patient with trapezius muscle due to long term postural changes.
multiple sclerosis who presents with a primary deficit of B. strengthening of the upper trapezius and pectoralis
dysmetria? major muscles for early thoracic outlet syndrome
A. isokinetic training using low resistance at fast symptoms.
movement speeds. C. stretching of scalenes and sternocleidomastoid
B. PNF patterns using carefully graded resistance and muscles for early thoracic outlet syndrome symptoms.
slow reversals. D. stretching of the pectoralis major and rhomboid
C. pool therapy using moderate water temperatures. muscles due to long term postural changes.
D. weight cuffs to distal extremities during functional
training. B10 C8 2
10. A 68 year-old patient with diabetes is exercising in your P.T.
B5 C6 3 clinic. He reports feeling weak, dizzy, and somewhat
5. A 16 year old patient suffered fractures of C4 and C5 nauseous. You notice that he is sweating profusely and is
following trauma received in a motor vehicle accident. unsteady on his feet. He then becomes confused about
Maximum stabilization of his cervical spine can BEST be where he is. You determine his problems are most likely
achieved with: due to and your immediate course of action should be:
A. four-poster orthosis. A. an insulin reaction; you call for emergency services.
B. halo orthosis. B. fatigue; you insist that he rest until his symptoms
C. Milwaukee orthosis. resolve.
D. soft collar. C. hyperglycemia; you have a nurse administer an
insulin injection.
B6 C3 2 D. hypoglycemia; you immediately administer orange
juice and notify his physician.

401936186.doc
2 Practice Questions

B11 C2 2 B16 C6 2
11. A patient with postpolio syndrome has started attending an 16. You determine that a patient is walking with a backward
aerobic exercise program as an outpatient. When she does trunk lean as he takes full weight on his right leg. He also
not show up for her scheduled session, you telephone to demonstrates great difficulty going up ramps. The BEST
find out what is the matter. She tells you that she is very intervention to remediate his problem is to:
tired and has not been able to get out of bed for the last A. strengthen hip extensors through bridging.
two days. Her muscles ache and she is afraid to continue B. strengthen knee extensors with weights, using 80%
with the exercise class. You decide to: one repetition maximum.
A. discharge her from the program since exercise is C. stretch hip abductors through sidelying positioning.
counterproductive in postpolio syndrome. D. stretch hip flexors through prone lying.
B. modify her exercise prescription to decrease the
frequency to once a week for an hour session, B17 C4 2
keeping the intensity moderate. 17. If a patient has developed a thick eschar secondary to a full
C. modify her exercise prescription to decrease the thickness burn, the antibacterial agent MOST effective for
intensity and duration but maintain a frequency of 3 infection control is:
times/week. A. silver nitrate.
D. reschedule her workouts for early morning when she B. Sulfamylon.
feels refreshed. C. Travase.
D. nitrofurazone.
B12 C7 3
12. You are the primary clinical instructor for a final affiliation B18 C8 1
physical therapy student. As you work with this student you 18. An 89 year-old has gradually lost much of his functional
become aware that she tends to process information all at vision over the last 4 years. He complains his vision is
once, not in an ordered step-by-step manner. Your best “foggy” and he has difficulty reading. He mistakes images
strategy to ensure adequate learning for this student is to: directly in front of him, especially in bright light. He does
A. focus the student on learning important relationships better walking across a room and is able to locate items in
and concepts. his environment with his peripheral vision when items are
B. focus the student on objective information and located to both sides. Based on his descriptions, you
interrelationships. suspect he may be presenting with:
C. provide real-life examples that link learning to her A. cataracts.
personal experiences. B. glaucoma.
D. redirect her to process information in a step-by-step C. homonymous hemianopsia.
manner. D. tunnel vision secondary to overdoses of steroid
medications.
B13 C7 2
13. A 69 year-old male falls and breaks his hip. He undergoes B19 C2 2
a total hip replacement at the local medical center. The 19. Following a traumatic brain injury, a 26 year-old woman is
physician orders physical therapy two days after surgery. inconsistently oriented to time and place. She is unable to
The patient is covered by Medicare and is still a patient in remember recent events and shows little or no carry-over
an acute care facility. The appropriate frequency of for new learning. The PRIMARY goal of rehabilitation at
treatment would be: this stage of recovery is to promote:
A. as often as possible, for the first two weeks based on A. an environmental and daily structure in which the
the therapist’s caseload. patient is best able to process stimuli cognitively.
B. daily, following the initial examination. B. functional independence in bed mobility and
C. every other day. transfers.
D. three to four times per day to facilitate early C. increased arousal and attention through the use of
discharge. sensory stimulation techniques.
B14 C3 1 D. independence in problem solving skills in variable
14. A patient is taking a drug from the sympathomimetic group, environments.
Albuterol. Which is NOT an expected effect of this drug? B20 C2 2
A. bronchodilation. 20. A 65 year-old woman suffered a right CVA one month ago.
B. exercise-induced bronchospasm. She demonstrates moderate tone in her left upper
C. hypertension extremity (predominantly flexor tone). Her major problem at
D. tachycardia. this time is a lack of voluntary movement control and her
left upper extremity shows minimal active movement with
B15 C5 2 1/4 inch subluxation of the shoulder. An appropriate initial
15. A 48 year-old female patient is recovering from a total knee activity would be:
replacement. Following her surgery, you receive a referral A. PNF D2 flexion pattern, left upper extremity.
to teach therapeutic exercises and a home program. The B. quadruped, rocking from side to side.
intervention not appropriate during the early postoperative C. sitting, left active shoulder protraction with extended
phase is: arm.
A. ankle pumps. D. sitting, weight bearing on extended left arm, weight
B. isotonic exercises using ankle weights. shifting.
C. seated knee extension.
D. straight leg raising. B21 C7 3

401936186.doc
Practice Questions 3

21. A therapist conducts a study of the effectiveness of hot and D. survey your audience just before the scheduled
cold in treating patients with pain. He recruits 200 patients session.
for each treatment group. The pain instrument he uses has
a possible total score of 50, with 50 being the worst pain. B27 C2 3
His data analysis reveals that Group A (heat modalities) 27. The patient with left hemiplegia would be least likely to
has a mean score of 33 with a standard deviation of 1.0 respond in therapy if the motor learning strategies
while Group B (cold modalities) had a mean of 35 with a emphasized:
standard deviation of 6.0. Based on this data the A. encouragement of the patient to slow down.
conclusion one should reach is: B. maximum use of demonstration and gesture.
A. cold has a greater effect on pain relief than heat. C. maximum use of verbal cues.
B. heat has a greater effect on pain relief than cold. D. simplification/restructuring of the environment
C. the spread of scores with cold treatment including removal of all clutter.
demonstrates variability is greater.
D. the spread of scores with heat treatment B28 C3 2
demonstrates variability is greater. 28. A two-week old infant born at 27 weeks gestation with
hyaline membrane disease is referred for a physical
B22 C1 1 therapy consult. Nursing reports that the child “desaturates
22. The therapist in the photograph is testing which muscle? to 84% with handling” and has minimal secretions at
A. anterior deltoid. present. The physical therapist should:
B. middle deltoid. A. delegate to a physical therapist assistant for a
C. supraspinatus. maintenance program of manual techniques for
D. upper trapezius. secretion clearance.
B. perform manual techniques for secretion clearance, 2
B23 C6 2 to 4 hours daily, to maintain airway patency.
23. A patient presents with pain and instability of the left C. provide suggestions to nursing for positioning for
foot/ankle secondary to poliomyelitis with more recent optimal motor development.
development of progressive post-polio muscle atrophy. In D. put the PT consult on hold as the child is too ill to
this case, a plastic solid ankle-foot orthosis is an tolerate exercise.
appropriate prescription in order to:
A. control excessive amounts of knee flexion during B29 C1 2
swing. 29. A thirty-four year-old male sustained a valgus stress to his
B. maintain dorsiflexion throughout swing. left knee while skiing. His orthopedist found a positive
C. provide modest assistance to dorsiflexion while McMurray’s test, and a positive Lachman stress test. He
restricting plantar flexion. has been sent to physical therapy for conservative
D. restrict all movement. management of his dysfunction. The sub-acute phase of
physical therapy intervention should emphasize:
B24 C1 1 A. closed-chain functional strengthening of the
24. A patient is referred to physical therapy following an quadriceps femoris and hamstrings emphasizing
anterior dislocation of the right shoulder. A possible positive regaining terminal knee extension.
examination finding as the result of this trauma would be: B. closed-chain functional strengthening of the
A. positive drop arm test. quadriceps femoris and hip abductors to promote
B. positive Neer test. regaining terminal knee extension.
C. weak deltoids. C. open-chain exercises of the hip extensors and
D. weak rhomboids. hamstrings to inhibit anterior translation of the femur
on the tibia.
B25 C8 1 D. open-chain strengthening of the quadriceps femoris
25. A 3 month-old infant is being examined at an early and hip adductors to inhibit anterior translation of the
intervention program. The therapist is having difficulty tibia on the femur.
flexing the right upper extremity to remove the infant’s
clothing. This task is made more difficult if: B30 C8 1
A. the infant is turned into sidelying on the left. 30. The most enthusiastic visual tracking by a neonate would
B. the infant sits up with support of the head in a neutral be elicited by:
position. A. a black and white face, with a red nose, held
C. the infant’s head is turned to the left. approximately nine inches from the infant’s eyes,
D. the infant’s head is turned to the right. moved horizontally.
B26 C7 3 B. a multicolored clown’s face, held 12 inches away and
26. You have been asked to give an inservice presentation to moved horizontally.
staff nurses on safe guarding techniques. In order to best C. a multicolored spinning top placed 15 inches from the
prepare for this talk, you should: infant’s eyes.
A. provide a questionnaire to a random sampling of D. a red ring on a string, held six inches away from the
participants one week before the scheduled infant’s eyes and moved horizontally.
presentation.
B. provide a questionnaire to all participants two weeks B31 C2 2
before the scheduled session. 31. A 14 year-old patient with traumatic brain injury has a
C. survey your audience at the scheduled session. convulsive seizure during a therapy session. She loses

401936186.doc
4 Practice Questions

consciousness and presents with tonic-clonic convulsions C. two electrodes with current flow parallel to the spinal
of all extremities. Your BEST response is to: column.
A. initiate CPR immediately and call for help to restrain D. two electrodes with current flow perpendicular to the
her. spinal column.
B. position in sidelying, check to see if the airway is
open, and immediately call for emergency assistance. B37 C8 1
C. position in supine with head supported with a pillow 37. An infant has just begun to pull-to-stand through kneeling,
and wait out the seizure. still demonstrates plantar grasp in standing, and is
D. use straps to secure her limbs so she can’t hurt independent in sitting including all protective extension
herself. reactions. This infant’s chronological age is approximately:
B32 C7 2 A. 5-6 months.
32. A therapist wants to know whether NDT handling B. 7-8 months.
techniques produces an improvement in independent C. 9-10 months.
rolling that lasts longer than 30 minutes. In this study rolling D. 12-14 months.
is the: B38 C1 2
A. control variable. 38. A sixty-five year old male retired bus driver has an
B. dependent variable. increasing frequency of low back pain over the last 10
C. independent variable. years. He states that NSAIDs help to relieve his symptoms
D. intervening variable. but there is always a nagging type pain. He reports
significant stiffness in the morning which dissipates by
B33 C3 1 noon after exercising and walking his dog. Pain is
33. A 45 year-old computer programmer, with no significant exacerbated with frequent lifting and bending activities as
past medical history, presents to the emergency room with well as sitting for long periods. Physical therapy
complaints of fever, shaking chills and a worsening intervention should emphasize:
productive cough. He has chest pains over the posterior A. joint mobilization, soft tissue mobilization, and flexion
base of his left thorax which is made worse on inspiration. exercises.
Which of the following is NOT an appropriate physical B. modalities to reduce pain, joint mobilization, and
finding for this patient? lumbar extension exercises.
A. asymmetrical breathing. C. modalities to reduce pain, postural reeducation and
B. crackles over the posterior aspect of his left thorax. dynamic stabilization exercises.
C. limited chest excursion. D. postural reeducation, soft tissue mobilization, and
D. slowed respiratory rate. dynamic stabilization.

B34 C6 2 B39 C7 3
34. A patient is demonstrating genu valgum during standing 39. You are orienting a new physical therapy aide in transfer
and walking This problem can be effectively controlled by techniques. Your initial consideration should be to:
prescribing a knee-ankle-foot orthosis with: A. ask about previous work and other experiences
A. anterior knee cap strap. related to transferring individuals.
B. posterior plastic shell. B. determine specific goals for teaching the techniques.
C. pretibial and suprapatellar anterior bands. C. give a computer simulated instructional program
D. quadrilateral brim. before actual “hands on” training takes place.
D. provide an organized series of talks dealing with
B35 C4 1 patient safety during transfers.
35. You are called in to consult on a nursing home patient who,
following a severe stroke and congestive heart failure, is B40 C7 2
confined to bed and is immobile. During a skin inspection 40. A 77 year-old patient with diabetes and bilateral lower
you note an area of erythema over her left sacrum which extremity amputation is to be discharged from an acute
persists longer than 50% of time pressure was applied to it. care hospital two weeks post surgery. The incisions on his
You determine that this is MOST likely: residual limbs are not healed and continue to drain. He is
A. an indurated decubitus ulcer with signs of maceration. unable to transfer because the venous graft sites in his
B. poor circulation secondary to end stage congestive upper extremities are painful and not fully healed. The
heart failure. MOST appropriate discharge destination for this patient
C. Stage 1 ulcer formation. would be:
D. Stage 2 ulcer formation. A. custodial care facility.
B. home.
B36 C5 3 C. rehabilitation hospital.
36. Your patient strained the lower back muscles three weeks D. skilled nursing facility.
ago. The patient complains of pain (6/10) and you note
bilateral muscle spasm from T11-L4. You elect to apply B41 C2 1
interferential current. The BEST electrode configuration to 41. A 54 year-old man awoke one morning with drooping left
choose in this case would be: facial muscles and excessive drooling. He was recovering
A. four electrodes with current flow diagonal to the spinal from a cold and had experienced an earache in his left ear
column. during the previous 2 days. You suspect a Bell’s Palsy
B. four electrodes with current flow perpendicular to the which can be confirmed by examining:
spinal column. A. corneal reflex and stretch reflexes of facial muscles.

401936186.doc
Practice Questions 5

B. taste sensation over the anterior tongue as well as foot and big toe. This is driving him crazy because he
motor function of facial muscles. knows the limb is gone. You recognize the source of his
C. taste sensation over the posterior tongue as well as discomfort is most likely pressure from residual limb
motor function of masseter muscle. wrapping affecting the:
D. trigger points for pain, especially over the TMJ joint. A. common peroneal nerve.
B. medial calcaneal nerve.
B42 C6 1 C. sural nerve.
42. You assign a physical therapist assistant to ambulate a 72 D. tibial nerve.
year-old patient with Parkinson’s disease. You instruct the
PTA to watch for: B48 C7 2
A. an abnormally wide base of support. 48. A therapist investigated the accuracy of pulse oximeter
B. decreased trunk rotation with shorter steps. estimates during exercise. Correlational analysis measured
C. unsteady, uneven gait with staggering steps to one the strength of the relationship between two types of ear-
side. probe-equipped pulse oximeters during heavy cycle
D. wider strides and increased double support time. exercise under hypoxic conditions. The investigator found
measured arterial oxyhemoglobin saturation (%HbO2)
B43 C3 1 levels to have a correlation of .89 at high saturation but
43. A patient with congestive heart failure is on a regimen of only .68 at low saturation levels. The results of this study
diuretics and calcium channel blockers. The potential side- suggest:
effects of these medications that the physical therapist A. accuracy of the measurements increases at higher
should be alert for include: saturation levels.
A. decreased electrolytes and electrical instability B. both machines are highly accurate at all saturation
evidenced by increased arrhythmias. levels.
B. gastrointestinal upset and extreme fatigue. C. both machines are only moderately accurate.
C. orthostatic hypotension and dizziness. D. during heavy exercise, oxygen saturation levels
D. reflex tachycardia and unstable blood pressure. should be interpreted cautiously.
B44 C1 2
44. A 42 year-old patient with a traumatic injury to her right B49 C1 1
hand had a flexor tendon repair to the fingers. Physical 49. A twenty-one year old college soccer player sustained a
therapy intervention following this type of repair should hyperextension knee injury when kicking the ball with his
begin: other lower extremity. The patient was taken to the
A. within a few days following surgery to preserve emergency room of a local hospital and was diagnosed
tendon gliding. with “knee sprain”. He was sent to physical therapy the
B. within a few days following surgery to allow for early next day for aggressive rehabilitation. As part of the
initiation of strengthening exercises. examination to determine the type of treatment plan to
C. after the splint is removed in 2-3 weeks to allow full implement, the therapist conducted the test shown in the
active range of motion of all affected joints. figure. Based on the test picture, the therapist is examining
D. after the splint is removed in 4-6 weeks to allow the integrity of the:
ample healing time for the repaired tendon. A. anterior cruciate ligament.
B. medial collateral ligament.
B45 C8 1 C. medial meniscus.
45. An 89 year-old community dwelling elder fell in her home D. posterior cruciate ligament.
and suffered multiple fractures of her right arm including B50 C1 2
Colles’ fracture of her right wrist, and humeral fracture of 50. A 28 year-old male week-end athlete sustained a right
her right shoulder. She is hospitalized for open reduction, shoulder injury when he was hit from behind while throwing
internal fixation of her right radius. Which is NOT an a football. His right upper extremity was flexed, abducted
expected finding with this patient? and externally rotated in preparation for throwing when he
A. decreased pain and tenderness at the fracture sites. was hit. Early physical therapy intervention should focus
B. increased likelihood of developing complications like on:
pneumonia. A. AROM which emphasizes abduction and external
C. mental confusion following hospitalization and rotation.
surgery. B. pendulum exercises.
D. slower healing time with prolonged rehabilitation. C. strengthening of the middle trapezius muscle.
D. strengthening of the rotator cuff muscles.
B46 C2 1
46. A patient exhibits bitemporal hemianopsia. In order to B51 C5 1
produce this condition, the lesion must be located at the: 51. You are to immerse an acutely sprained ankle into an ice
A. left optic radiation. water bath. You should tell the patient to FIRST expect:
B. left temporal lobe. A. aching, numbness, and burning, followed by intense
C. optic chiasm. cold.
D. right parietal lobe. B. burning, intense cold, and aching, followed by
numbness.
B47 C6 1 C. intense cold, burning, and aching, followed by
47. Your patient is recovering from a left tibial amputation and numbness.
complains of numbness and tingling affecting his dorsal D. numbness, aching, and intense cold, followed by
burning.

401936186.doc
6 Practice Questions

B52 C8 3 D. using armrests on the chair.


52. Upon admission to a local hospital, a patient is given an
Advanced Care Medical Directive to review and sign. B57 C1 1
Guidelines for this document would NOT include: 57. A baseball pitcher has been sent to physical therapy with
A. an individual can refuse treatment even if it leads to progressive posterior shoulder pain and weakness of the
death. shoulder abductors and lateral rotators. You notice muscle
B. an individual must be mentally competent, with wasting superior and inferior to the scapular spine. The
signing witnessed by two adults. patient’s problem is MOST LIKELY attributable to damage
C. individual facilities modifying its format and involving the:
implementation schedule. A. long head of the biceps brachii.
D. patients, upon hospital admission, must be informed B. scalene muscles.
of their right to make decisions regarding their C. spinal accessory nerve.
medical care. D. suprascapular nerve.

B53 C2 2 B58 C7 3
53. Your patient is recovering from stroke. At 4 months he is 58. Nursing homes that receive Medicare reimbursement for
ambulating with a straight cane for household distances. In eligible residents are required by law to provide for
the clinic, when you take his cane away and have him rehabilitation services including physical therapy based on:
practice ambulating with no assistive device, you observe A. diagnostic categories (DRGs).
recurvatum that increases as the session continues. The B. needs assessment performed by a physical therapist.
BEST strategy for you to take is to: C. referral from a physician.
A. give him a KAFO to control the hyperextension and D. referral from the nurse case manager.
have him use a hemi walker.
B. give him a small based quad cane to improve his B59 C2 2
stability and have him practice AROM in supine. 59. A 62 year-old patient is recovering from surgical resection
C. practice isolated small range quadriceps eccentric of an acoustic neuroma. She presents with symptoms of
control work in standing and continue with the straight dizziness, vertigo, horizontal nystagmus, and postural
cane. instability. To address these problems, her physical therapy
D. put him on a Cybex and work on increasing plan of care should incorporate:
quadriceps torque output at higher loads and A. Hallpike exercises to improve speed in movement
increasing speeds. transitions.
B. prolonged bed rest to allow vestibular recovery to
B54 C6 2 occur
54. Your patient tells you that he is having difficulty going down C. repetition of movements and positions that provoke
ramps. He reports his knee wobbles and seems unsteady. dizziness and vertigo.
An appropriate intervention for this problem would be: D. strengthening exercises focusing on spinal extensors.
A. biofeedback training to reduce a knee extensor
spasticity. B60 C6 1
B. progressive resistance training for the quadriceps. 60. Your patient is beginning her ambulation training with a
C. prolonged icing to reduce hamstring pain. right above-knee prosthesis. You notice that during early
D. stretching via a posterior resting splint for tight plantar swing the heel rises excessively. Possible causes are:
flexors. A. amputee pain or discomfort.
B. inadequate knee friction or too little tension in the
B55 C7 3 extension aid.
55. It is illegal to release patient information without obtaining C. too much knee friction or too little tension in the
the patient’s consent to: extension aid.
A. another healthcare provider involved in the care of D. too much tension in the extension aid.
the patient.
B. the insurance company that is paying for the patient’s B61 C4 2
treatment. 61. You are treating an elderly 93 year-old in the home
C. the patient’s employer when the condition is work- environment. When you arrive you note that he is
related. somewhat confused and his color and skin turgor is poor.
D. the referring physician when there is a referral He reports that he has had an intestinal bug the last few
relationship. days with frequent vomiting and diarrhea. Your BEST
course of action is to:
B56 C6 3 A. cancel therapy for today and carefully document your
56. With respect to the worker’s sitting postures in the Figure, findings in his chart.
the greatest reduction in lumbar spine compression forces B. give him some water to drink and report your findings
would be achieved by: (2002) The Kinematics of Sitting, to his physician immediately.
Herman Miller Co., with permission C. monitor his vital signs as you get him up and
A. combining the usage of a lumbar support, armrests, ambulating.
and increase the angle between the seat pan and D. notify the family, request they keep him company until
backrest to between 90-110 degrees. he gets over his illness.
B. reclining the backrest of the chair at an angle
between 90-110 degrees. B62 C3 2
C. using a lumbar support.

401936186.doc
Practice Questions 7

62. For patients with coronary artery disease participating in a A. foot is inset too much.
cardiac rehabilitation class an indicator that the exercise B. foot is outset too much.
intensity is too great and should be reduced would be: C. socket is aligned too far back or tilted posteriorly.
A. a systolic blood pressure greater than 140 mm Hg or D. socket is aligned too far forward or tilted anteriorly.
diastolic BP greater than 80 mm Hg.
B. appearance of a PVC on the ECG. B68 C3 2
C. greater than 1 mm ST segment depression, horizontal 68. A chronic smoker developed carcinoma of the lung.
or downsloping. Following a right pneumonectomy, post-operative physical
D. peak exercise HR greater than 140. therapy intervention should include:
A. breathing exercises to both sides of the thorax to
B63 C8 2 maintain adequate aeration.
63. An infant who was 33 weeks gestational age at birth and is B. percussion in all postural drainage positions.
now 3 weeks chronological age demonstrates colic. In this C. shaking in all postural drainage positions.
case, the BEST intervention the physical therapist could D. sustained maximal inspiration training with an
teach the mother is: incentive spirometer.
A. fast vestibular stimulation.
B. neutral warmth. B69 C6 2
C. stroking and tapping. 69. As part of your plan of care, you have to order a wheelchair
D. visual stimulation with a colored object. for a 32 year-old patient with a T9-10 spinal cord injury. Her
wheelchair won’t need:
B64 C2 1 A. a high back.
64. You are treating a 24 year-old man who exhibits Brown- B. a low back.
Sequard syndrome as a result of a gun shot wound. Your C. removable arm rests.
examination should reveal: D. swing-away footrests.
A. ipsilateral weakness and loss of position sense and B70 C1 2
vibration below the lesion level with contralateral loss 70. A patient complains of persistent wrist pain after painting
of pain & temperature sensation. her house three weeks ago. The patient demonstrates a
B. loss of motor function and pain and temperature positive Finkelstein’s Test, pain and swelling at the
sensation with preservation of light touch and position anatomical snuff box, and decreased pinch strength. An
sense below the level of the lesion. effective plan of care should include:
C. loss of upper extremity function, (cervical tract A. desensitization techniques to reduce symptoms of
involvement) with preservation of lower extremity reflex sympathetic dystrophy.
function (lumbosacral tract involvement). B. gentle active assistive exercises to regain function
D. sparing of tracts to sacral segments with preservation after sustaining a gamekeepers thumb injury.
of perianal sensation, and active toe flexion. C. iontophoresis to reduce inflammation at the carpal
tunnel.
B65 C7 2 D. phonophoresis to reduce inflammation of the
65. A 62 year-old man is recovering from stroke and presents tendinous sheaths.
with moderate impairments of the left upper and lower
extremities. Your goal today is to instruct him in a stand- B71 C8 2
pivot transfer to his affected side so he can go home on a 71. An adaptive wheelchair for a child with moderate spastic
weekend pass. His wife is attending today’s session and quadriplegic cerebral palsy would include:
will be assisting him on the weekend. Your BEST choice for A. a pommel to keep hips abducted.
instruction is: B. movable chairback to allow for extension of the hips
A. demonstrate the task, then have the caregiver and trunk.
practice with the patient. C. movable footrests to allow ankles to plantarflex.
B. demonstrate the task, then practice with the patient. D. pads on the seat to keep hips adducted.
C. practice first with the caregiver, then with the patient.
D. practice first with the patient, then with the caregiver. B72 C2 1
72. A 73 year-old has persistent balance dysfunction and a
B66 C7 3 history of recent falls (2 in the last 3 months). In your initial
66. Your patient is a 78 year-old male currently in an acute session it is crucial to examine:
care facility recovering from fractures of the spine and hip. A. cardiovascular endurance during a 6 minute walking
His Medicare Part A benefits have come to an end and he test.
is soon to be discharged. Medicare Part B will NOT cover: B. level of dyspnea during functional transfers.
A. durable medical equipment needed for the patient in C. sensory losses and sensory organization of balance.
his home. D. spinal musculoskeletal changes secondary to
B. home health agency rehabilitation services. degenerative joint disease.
C. outpatient therapy services in a private therapy
practice. B73 C7 3
D. outpatient therapy services in an acute care facility. 73. A patient with adhesive capsulitis of the shoulder sustains a
fracture of the shoulder during treatment provided by the
B67 C6 1 PTA. The fracture occurred while the PTA was mobilizing
67. When using a patellar-tendon bearing prosthesis, a patient the shoulder joint, which was part of the plan of care
will experience excessive knee flexion in early stance if the: established by the PT. After the incident, the PTA told the

401936186.doc
8 Practice Questions

PT that she was not familiar with the mobilization C. take his BP pre and post exercise, using his non-
techniques to the shoulder. Responsibility in this case falls shunt arm.
on: D. wait until he stops walking before taking his BP in the
A. both the PT and the PTA are responsible for seated position.
establishing effective communication regarding the
skills and competencies of the PTA. B78 C6 2
B. neither the PT nor the PTA are responsible for the 78. A patient is recovering from a fracture of both tibia and
fracture, it is an accepted risk associated with joint fibula in the right lower leg which has been casted. The
mobilization. referral is for gait training, nonweightbearing on right lower
C. the PT who is solely responsible for assessing the extremity. The safest gait pattern for this patient to use is a:
competence of the PTA under their supervision. A. four-point.
D. the PTA who is responsible for informing their B. swing-to.
supervising PT whenever they are unfamiliar or C. three-point.
uncomfortable with any treatment procedure. D. two-point.

B74 C7 3 B79 C7 2
74. A new patient arrives at the physical therapy department 79. A therapist wishes to determine the effectiveness of TENS
for an initial examination, evaluation and treatment of a on the relief of pain in a group of 20 patients with phantom
second degree ankle sprain. The therapist is busy writing a limb pain. She recruits her patients over a 2 year period. All
discharge summary when the patient with the ankle sprain receive a 6 week intervention. She finds that 12 patients
arrives for his scheduled appointment. The physical with phantom pain got pain relief while 8 had no relief. The
therapist assistant on staff knows the patient as he had BEST conclusion that can be reached is:
treated the patient previously for a similar injury. The A. 40% of patients with phantom pain do not benefit
physical therapist should: from TENS.
A. ask the physical therapist assistant to evaluate the B. since a no treatment group was not used for
patient’s range of motion and functional status to comparison, no conclusions can be reached.
speed up the evaluation process. C. since a quasi-experimental design was used, the
B. ask the physical therapist assistant to perform a treatment effectiveness of TENS is established.
complete evaluation. D. TENS has a 60% rate of effectiveness.
C. complete the written component of the discharge
summary at another time and evaluate this new B80 C3 2
patient. 80. Following a post-myocardial infarction, a patient is a new
D. tell the patient to take a seat and wait until the admission to your Phase 3 cardiac rehabilitation program.
discharge summary is completed. During the initial exercise session his ECG responses are
continuously monitored via radio telemetry. You notice
B75 C1 1 three PVCs occurring in a run with no P wave. The MOST
75. During a postural screen for a patient complaining of low appropriate action to take is to:
back pain you notice that the knees are in genu A. continue the exercise session but monitor closely.
recurvatum. Possible contributory postures would include: B. have him sit down and rest for a few minutes before
A. ankle dorsiflexion and hip abduction. resuming exercise.
B. ankle plantarflexion and anterior pelvic tilt. C. modify the exercise prescription by decreasing the
C. forefoot varus and posterior pelvic tilt. intensity.
D. lateral tibial torsion and anterior pelvic tilt. D. stop the exercise and notify the physician
B76 C1 1 immediately.
76. After treating a patient for trochanteric bursitis for one
week, the patient has no resolution of pain and is B81 C2 1
complaining of problems with gait. After re-examination you 81. A 63 year-old patient with left hemiplegia is able to
find weakness of the quadriceps femoris, and altered recognize his wife after she is with him for awhile and talks
sensation at the greater trochanter. You suspect the to him, but he is unable to recognize the faces of his
problem is: children when they come to visit. The children are naturally
A. degenerative joint disease of the hip. very upset by their father’s behavior. The BEST
B. L4 nerve root compression. explanation for his problem is:
C. L5 nerve root compression. A. anosognosia.
D. sacroiliac dysfunction. B. ideational apraxia.
C. somatognosia.
B77 C4 1 D. visual agnosia.
77. You are treating a patient in chronic renal failure. You need B82 C8 1
to schedule his physical therapy sessions around his 82. You have received a referral to evaluate the fall risk
dialysis which he receives 3 mornings a week. He is also potential of an 82 year-old community dwelling elder. He
hypertensive and requires careful monitoring during his has fallen three times in the last four months with no history
ambulation program. Your BEST approach is: of fall injury except for minor bruising. He currently
A. place him supine before taking his BP, using the presents with an increased fear of falling. The factor
shunt arm. LEAST likely to contribute to his fall risk is:
B. take his BP every minute during exercise, using his A. dementia or depression.
shunt arm. B. drugs that produce essential tremor.
C. drugs that result in orthostatic hypotension.

401936186.doc
Practice Questions 9

D. psychotropic drugs. finger motion, and dry scaly skin over the involved areas.
The physical agent MOST appropriate to select in this case
B83 C1 2 would be:
83. After examining a patient with foot pain, the therapist finds A. functional electrical stimulation, seated.
the following positive findings: reproduction of symptoms in B. hot packs with the patient positioned in a recumbent
weight-bearing and running on a treadmill, pes planus and position.
pain with palpation at the distal aspect of the calcaneus. C. infrared with the patient positioned supine on a
Late sub-acute management, when pain is treatment table.
manageable/reduced, would include: D. paraffin with the patient positioned in a recumbent
A. examination for an orthosis. position.
B. modalities to reduce pain and inflammation.
C. strengthening of ankle dorsiflexors. B89 C6 2
D. stretching of plantar fascia. 89. Your patient is 82 years-old and demonstrates a history of
recent falls (two in the last two months) and mild balance
B84 C6 3 instability. Your referral is to examine the patient and
84. A 22 year-old patient with a complete T10 paraplegia recommend an assistive device as needed. Based on the
resulting from a spinal cord injury is ready to begin patient’s history, it would be BEST to select a:
community wheelchair training. Today your goal is to teach A. a folding reciprocal walker.
him how to do a wheelie so he can manage curbs. Your B. front wheel rolling walker that folds.
BEST training strategy is to instruct the patient to: C. hemi walker.
A. grasp the handrims posteriorly and pull them forward D. standard, fixed frame walker.
abruptly and forcefully.
B. lean backward while moving the hands slowly B90 C1 3
backward on the rims. 90. Following surgery, a patient responds with a stiff pelvis and
C. place his hand on the top of the handrims to steady limited pelvic/lower trunk mobility. You elect to use sitting
the chair while he throws his head and trunk forward. exercises on a Swiss ball to correct these impairments. In
D. throw his head and trunk backward to rise up on the order to improve lower abdominal control, the ball would
large wheels. have to move:
A. backward, producing anterior tilting of the pelvis.
B85 C3 1 B. backward, producing posterior tilting of the pelvis.
85. A physical therapist should be alert to recognize the signs C. forward, producing anterior tilting of the pelvis.
and symptoms associated with the onset of aspiration D. forward, producing posterior tilting of the pelvis.
pneumonia. Of the following, the patient MOST susceptible
to develop this form of pneumonia is one with: B91 C2 2
A. a circumferential burn of the thorax associated with 91. A patient who has been in a coma for 8 weeks is newly
significant pain. admitted to an extended care facility. Your focus is to
B. a complete spinal cord lesion at T2 with diminished involve the family in the plan of care since both parents are
coughing ability. faithful in visiting on daily basis. You decide it is MOST
C. amyotrophic lateral sclerosis with dysphagia and important to:
diminished gag reflex. A. demonstrate PROM techniques, offering corrections
D. severe scoliosis with compression of internal organs. as they perform the exercises for you.
B. give them written instructions detailing his PROM
B86 C6 2 exercises.
86. A patient demonstrates quadriceps weakness (4- /5) and C. instruct them to talk to the patient but restrict them
difficulty descending stairs. The BEST intervention to from all hands on assistance.
regain functional strength in the quadriceps is: D. tell them their participation will come once the patient
A. isokinetic exercise, at 36 degrees/second. becomes conscious, right now the professional staff
B. maximum isometric exercise, at 45 and 90 degrees of must do his exercises.
knee extension.
C. partial squats progressing to lunges. B92 C2 1
D. progressive resistance exercises, 70% 1 RM, 3 sets 92. A patient presents with weakness of the entire right half of
of 10. the face. She is unable to completely close her eye.
Sensation is normal. She tells you that she recently had the
B87 C2 1 flu and has recovered slowly from this illness. You suspect:
87. Independent community ambulation as the primary means A. abducens nerve pathology: the right eye pulls inward.
of functional mobility is a realistic functional expectation for B. facial nerve pathology: Bell’s palsy.
a patient with the highest level of spinal cord injury at: C. oculomotor nerve pathology: nystagmus.
A. high lumbar (T12-L1). D. trigeminal nerve pathology: tic douloureux.
B. low lumbar (L4-L5).
C. low thoracic (T9-10). B93 C3 2
D. midthoracic (T6-T9). 93. A patient with bacterial pneumonia has crackles and
B88 C5 2 wheezes at her left lateral basal segment and decreased
88. Your patient had multiple fractures of both hands and wrists breath sounds throughout. She in on 4 liters of oxygen by
as a result of a mountain bike accident. Now five weeks nasal cannula which brings her SaO2 to 90%. Respiratory
later, the patient currently has vertigo, limited wrist and

401936186.doc
10 Practice Questions

Rate is 28. The intervention that would be B. hand splints (intrinsic minus position) with plastic
INAPPROPRIATE to use in this case is: jacket for the trunk.
A. breathing exercises encouraging expansion of the left C. plastic cervical orthosis and axillary splints (airplane
lateral basilar thorax. position).
B. percussion to the appropriate area on the left lateral D. soft cervical collar that allows some neck flexion.
basilar thorax.
C. shaking over the appropriate area on the left lateral B99 C1 1
basilar thorax. 99. A 16 year-old female, seen in the emergency room, was
D. standard postural drainage for the lateral basal placed in a short leg cast following a severe right-sided
segment, left lower lobe. Grade III ankle sprain. She is immediately referred to
B94 C8 1 physical therapy for gait training using crutches. She
94. You receive a referral to see an elderly patient in the ICU complains that her right great toe is numb and she cannot
recovering from a severe case of pneumonia. You lift her toes. This problem MOST LIKELY is a result of:
recognize his disorientation is due to delirium rather than A. anxiety and hyperventilation in anticipation of using
dementia because: crutches.
A. he demonstrates persistent personality changes. B. compression of the common peroneal nerve.
B. he has hallucinations throughout the day. C. compression of the medial plantar nerve.
C. his level of arousal is significantly depressed. D. compression of the posterior tibial artery resulting in
D. this cognitive symptoms are intermittent and come on vascular insufficiency.
suddenly, often at night.
B100 C2 3
B95 C7 2 100. Which intervention would be LEAST likely used to improve
95. A comparison of the effects of exercise in water, on land, or left-sided neglect in a patient with left hemiplegia?
combined on the rehabilitation outcome of patients with A. bridging with arms at sides.
intra-articular anterior cruciate ligament reconstructions B. hooklying, lower trunk rotation, rhythmic initiation.
revealed that less joint effusion was noted after 8 weeks in C. rolling, supine to sidelying on right, using PNF lift
the water group. An appropriate statistical test to compare pattern.
the girth measurements of the three groups is: D. sitting, with hands forward resting on large ball,
A. analysis of covariance. weight shifting moving ball to left.
B. analysis of variance.
C. chi square. B101 C7 3
D. Spearman rho. 101. A 20 year-old patient with traumatic brain injury who has
been receiving in-patient physical therapy for the past two
B96 C2 2 months has not demonstrated improvement in functional
96. A 64 year-old man has a recent history of strokes (2 in the status for a considerable period of time. The patient is
past 4 months). He demonstrates good return of his right covered by Medicare. You have informed both the referring
lower extremity and is walking with a straight cane. You are physician and the patient’s family of his lack of progress.
concentrating on improving his balance and independence The family insists that you continue to treat the patient, and
in gait. Unfortunately his speech recovery is lagging behind the physician continues to refer the patient for more
his motor recovery. He demonstrates a severe fluent treatment and certifies the necessity of the care. You
aphasia. The BEST strategy during therapy is: should:
A. consult with the speech therapist to verbally interpret A. continue to provide the care both the family and the
your instructions. referring physician demand; it is the physician’s
B. involve the family for all treatment sessions and have responsibility to determine the appropriateness of
them translate for you. physical therapy.
C. utilize demonstration and gesture to get the idea B. give the patient’s family Medicare notification of
across of what you want him to do. noncoverage information, and carefully explain it, and
D. utilize verbal cues, emphasizing consistency and their options, which could include paying for the care
repetition. out-of-pocket.
C. modify treatment goals in a manner that will allow you
B97 C6 1 to demonstrate that the treatment is achieving
97. An individual is walking with an above-knee prosthesis and progress towards reasonable goals.
demonstrates terminal swing impact. You suspect: D. refer the patient to another clinic that you believe will
A. insufficient knee friction. be willing to continue treatment despite lack of
B. the hip flexors are weak. functional improvement.
C. the prosthesis is externally rotated.
D. too little tension in the extension aid. B102 C3 2
102. A patient with COPD reports to his fourth outpatient
B98 C4 2 pulmonary rehabilitation session complaining of nausea,
98. A 52 year-old man was burned over 40% of his body in an gastric upset and feeling jittery. The patient reports no
industrial accident. He has full thickness burns over his change in pulmonary symptoms. The physical therapist
anterior trunk and neck and superficial partial thickness records the following set of vital signs: temperature 98.6°F,
burn over his shoulders. The MOST appropriate splinting heart rate 110 beats/min, and irregular blood pressure
procedures to stabilize this patient out of positions of 150/86, respiratory rate 20. Breath sounds show no change
common deformity include: from baseline. The therapist checks the medical record and
A. arm slings that fasten the arms to the chest. finds that the patient has no history of gastric disease. He

401936186.doc
Practice Questions 11

is presently taking theophylline, Ventolin and Azmacort. and type of testing modality: arm ergometry versus leg
The physical therapist should: ergometry. She finds a correlation 0.59 with the arm testing
A. call the patient’s physician due to signs of while the correlation is 0.79 with the leg testing. She
theophylline toxicity. determines:
B. have the patient call the physician when he returns A. both arm and leg ergometry are highly correlated with
home due to signs of increased pulmonary RPE.
dysfunction. B. both arm and leg ergometry are only moderately
C. have the patient use his Ventolin to improve correlated with RPE.
respiratory status. C. leg ergometry is highly correlated with RPE while arm
D. send the patient home and reschedule for another ergometry is only moderately correlated.
day. D. the common variance of both types of testing is only
22%.
B103 C8 1
103. You are treating a one year-old child with Down Syndrome B108 C1 2
at home and you notice a decrease in strength in the 108. A 67 year-old male has a diagnosis of left knee
extremities. DTRs are 1+. Your immediate thought is: degenerative joint disease. He complains of left-sided knee
A. hypertonia and decreased strength are to be pain of two months duration. He has been followed by
expected with Down Syndrome. outpatient physical therapy for three weeks. The patient
B. hypotonia and decreased strength are to be expected feels his condition is worsening as pain has increased
with Down Syndrome. during weight bearing activities and he can no longer fully
C. the parents are not performing the home exercise extend his left knee. Examination findings include:
program. increased swelling, decreased knee AROM into extension,
D. there may be a dislocation at C1/C2 due to and an antalgic gait. The physical therapist should:
ligamentous laxity. A. continue physical therapy for another two weeks
because there is uncertainty if the patient
B104 C2 1 understands or is complying with the home exercise
104. Which of the following causative factors is NOT likely to program.
produce shoulder pain in hemiplegia? B. continue therapy for another week to ensure that all
A. flaccid paralysis with loss of seating action of rotator interventions have been attempted and then return
cuff muscles. the patient to the referring physician.
B. passive range of motion while emphasizing C. immediately return the patient to the referring
scapulohumeral rhythm. physician with documentation indicating that
C. spastic retraction with depression and downward treatment was ineffective.
rotation of scapula. D. tell the patient to see an orthopedic surgeon for
D. traction and gravitational forces acting on a possible immediate surgical intervention.
depressed, downwardly rotated scapula.
B109 C5 3
B105 C6 1 109. While playing volleyball two months ago, a 28 year-old had
105. While gait training a patient recovering from a cerebral an inversion sprain of the calcaneofibular and anterior
vascular accident, you observe the knee on the affected talofibular ligaments of the right ankle. The ankle is still
side going into recurvatum during stance phase. The painful, very limited in motion and slightly tender to the
MOST LIKELY cause of this deviation can be attributed to: touch. As part of your intervention, ultrasound treatment
A. severe spasticity of the hamstrings or weakness of parameters should consist of:
the gastrocnemius-soleus. A. continuous US @ 1 MHz.
B. weakness of both the gastrocnemius-soleus and B. continuous US @ 3 MHz.
pretibial muscles. C. pulsed US @ 1 MHz.
C. weakness of the gastrocnemius-soleus or spasticity D. pulsed US @ 3 MHz.
of the pretibial muscles.
D. weakness or severe spasticity of the quadriceps. B110 C2 3
110. In Neurodevelopmental Treatment (NDT) of the patient
B106 C4 2 recovering from stroke, therapy would NOT include:
106. A patient with an ulcer is treated with a whirlpool to assist in A. facilitation of early movement in synergistic patterns
debridement and wound cleansing. The laboratory report followed quickly by movement patterns out-of-
comes back positive for active local infection. You instruct synergy.
the physical therapist assistant to drain and clean the B. facilitation of selective movement control out of
whirlpool. Disinfection of the whirlpool can best be synergistic patterns.
achieved with: C. functional activities emphasizing reintegration of the
A. boiling water. hemiplegic side.
B. povidone-iodine. D. reduction of spasticity and abnormal reflex activity
C. silver nitrate solution. through positioning and handling techniques.
D. ultraviolet exposure.
B111 C8 2
B107 C7 2 111. A 4 year-old child with moderate spastic diplegia is referred
107. A therapist has completed a study investigating the to physical therapy for an adaptive equipment check. In this
relationship between ratings of perceived exertion (RPE) case, which equipment is NOT indicated?
A. bilateral KAFO’s.

401936186.doc
12 Practice Questions

B. posture control walker (posterior walker). B117 C6 2


C. prone stander 117. To correct for the problem of a forward festinating gait in a
D. tone reducing AFOs. patient with Parkinson’s disease, you could:
A. increase cadence.
B112 C6 2 B. increase stride length.
112. A 67 year-old patient is recovering from a left CVA. He is C. use a heel wedge.
wearing a plastic knee-ankle-foot orthosis to stabilize his D. use a toe wedge.
right foot. During gait analysis you observe lateral trunk
bending toward the right as he bears weight on the right leg B118 C6 3
at midstance. The BEST choice to correct this problem is: 118. A patient presents with difficulty with fast movement speeds
A. provide a lift on the shoe of the involved leg. and fatigues easily. You decide on a strength training
B. strengthen hamstrings on the right side. program that specifically focuses on improving fast-twitch
C. strengthen hip flexors on the right side. fiber function. The optimal exercise prescription to achieve
D. strengthen the hip abductors on the right side. this goal is:
B113 C1 2 A. high intensity workloads for long durations.
113. A ten year-old male who plays catcher on his baseball B. high intensity workloads for short durations.
team complains of bilateral knee pain which is exacerbated C. low intensity workloads for long durations.
with forceful quadriceps contraction. He has also noticed D. low intensity workloads for short durations.
pain and swelling at the distal attachment of the patellar
tendon. Effective early physical therapy intervention should B119 C2 1
include: 119. You have been called to the traumatic brain injury unit to
A. AROM exercises to prevent contracture. examine a new patient. The patient is highly agitated and
B. casting followed by decreased loading of the knee. combative with the nursing staff. You assess his current
C. decreased loading of the knee by the quadriceps state as being one of high arousal. The autonomic changes
femoris muscle. you would expect to see include:
D. modalities to decrease inflammation. A. decreased responsiveness to sensory stimulation and
lethargy.
B114 C3 2 B. hyperactive stretch and cutaneous reflexes,
114. An 83 year-old patient has a decubitus ulcer of 3 months exaggerated pupillary reflexes.
duration on his lateral ankle. The ankle is swollen, red, and C. increased HR and RR along with dilated pupils.
painful with a moderate to high amount of wound drainage D. slowed HR with constriction of pupils, hypotension.
(exudate). The BEST dressing for this wound is:
A. calcium alginate dressings. B120 C8 2
B. gauze dressings. 120. A physical therapy functional goal for a 5 year-old child with
C. hydrogel dressings. a very high lumbar lesion myelomeningocele and minimal
D. semipermeable film dressings. cognitive involvement would be;
A. ambulation for exercise with a reciprocating gait
B115 C1 1 orthosis and Lofstrand crutches.
115. A thirty-five year-old male was diagnosed with a bulging B. community ambulation with a reciprocating gait
disc at the right L5-S1 spinal level without nerve root orthosis and Lofstrand crutches.
compression. The functional loss he would MOST likely C. community ambulation with HKAFOs and Lofstrand
suffer from is: crutches.
A. centralized gnawing pain with loss of postural control D. physiological ambulation with KAFOs and rollator
during lifting activities. walker.
B. centralized gnawing pain with uncompensated
gluteus medius gait. B121 C7 2
C. radicular pain to the right great toe with a 121. A patient with multiple sclerosis is part of a national study
compensated gluteus medius gait. testing the effectiveness of a new medication. The patient
D. radicular pain to the right great toe with difficulty reports that the pill she is taking makes her feel much
sitting for long periods. better and allows her to move easier. At the conclusion of
the study it is revealed that she was part of the control
B116 C2 2 group. Her responses are MOST likely due to:
116. High level management of an individual recovering from A. hawthorne effect.
traumatic brain injury who demonstrates Rancho Level B. placebo effect.
Cognitive Function VII should focus on the avoidance of C. pretest-treatment interference.
“Robot syndrome.” The best intervention strategy in this D. sampling bias.
case is represented by:
A. involving the patient in decision making, emphasizing B122 C3 1
safety and independent performance. 122. You are reading a recent report of arterial blood gas
B. providing a high degree of environmental structure. analysis with the following values: Fraction of inspired
C. providing assistance for guided movements during all oxygen (FiO2) - 0.21, PaO2 - 53 mmHg, PaCO2 - 30
movement tasks. mmHg, pH - 7.48, Bicarbonate ion - 24 mEq/l This would
D. providing maximum supervision to ensure successful indicate that your patient is in:
performance. A. metabolic acidosis.
B. metabolic alkalosis.

401936186.doc
Practice Questions 13

C. respiratory acidosis. D. stretching of the pectoralis minor muscle following


D. respiratory alkalosis. acromioclavicular joint inflammation.

B123 C4 3 B128 C4 1
123. A 32 year-old patient is referred to your cardiac exercise 128. A 22 year-old male has a one year history of AIDS. The
group following a mild myocardial infarction. From her case worker reports a gradual increase in difficulty with
intake questionnaire you learn she also has Type I (IDDM) instrumental ADLs. A referral to P.T. is initiated. The most
diabetes, controlled with twice daily insulin injections. In likely CNS deficits from AIDS are:
order to minimize the risk of a hypoglycemic event during A. alterations in memory, confusion, and disorientation.
exercise, you should have the patient: B. gait disturbances resulting from progressive rigidity
A. avoid exercise during periods of peak insulin activity. and tremor.
B. exercise daily for 40-50 minutes to achieve proper C. increasing paralysis eventually leading to
glucose control. quadriplegia.
C. have the patient decrease her carbohydrate intake for D. pronounced sensory loss resulting in sensory ataxia.
2 hours before the exercise session.
D. monitor blood glucose levels carefully every week B129 C7 3
during the rehabilitation program. 129. A new graduate physical therapist has an appointment for a
job interview with Human Resources at a large teaching
B124 C8 2 hospital. She is well dressed, has a professional
124. An 85 year-old resident of a community nursing home is typewritten resume, and is prompt for her 2:00 p.m.
diagnosed with organic brain syndrome, Alzheimer’s type. appointment. The interviewer should AVOID discussing:
In formulating a plan of care, it is important to understand A. health insurance, continuing education, and vacation
that she: days.
A. can usually be trusted to be responsible for her daily B. information regarding advantages of working in a
care needs. large teaching hospital as well as disadvantages
B. can usually be trusted with transfers with appropriate regarding the job.
positioning of the wheelchair. C. the applicant’s marital status.
C. is more likely to remember current experiences then D. work hours associated with the position.
past ones.
D. will likely be resistant to activity training if unfamiliar B130 C8 3
activities are used. 130. You are working with a two year-old child with Down
Syndrome who frequently uses a W sitting position. You
B125 C2 3 are explaining to the parents why this position should be
125. A patient is having difficulty learning how to transfer from discouraged. The main reason to discourage W sitting in
mat to wheelchair. He just can’t seem to get the idea of this young child is that it may:
how to coordinate this movement. In this case, the MOST A. cause femoral anteversion and knee strain.
effective use of feedback during early motor learning B. decrease abnormally low tone.
should: C. delay the development of normal sitting.
A. focus on guided movement and proprioceptive inputs. D. increase abnormally high tone.
B. focus on knowledge of performance and
proprioceptive inputs. B131 C2 1
C. focus on knowledge of results and visual inputs. 131. A 92 year-old patient is being examined for signs of stroke.
D. provide feedback only after a brief (5 second) delay. When you test two-point discrimination on his right hand,
B126 C6 2 he is unable to tell you whether you are touching him with
126. A 52 year-old women presents with pain and paresthesias one or two points. You determine that there is impaired
affecting the third and fourth toes of her left foot. She function in the:
typically wears shoes with 3 inch heels and pointed toes. A. anterior spinothalamic tract or thalamus.
You suspect she is presenting with metatarsalgia. The B. dorsal column/lemniscal pathways or somatosensory
BEST intervention is a: cortex.
A. pad over the transverse arch. C. lateral spinothalamic tract or somatosensory cortex.
B. pad positioned distal to the metatarsal heads. D. spinal lemniscus or ventral posterolateral nucleus of
C. scaphoid pad. the thalamus.
D. Thomas heel. B132 C3 2
132. A 75 year-old outpatient has a history of two myocardial
B127 C1 2 infarctions and one episode of recent congestive heart
127. A 38 year-old female developed right throbbing shoulder failure. He also had claudication pain in the right calf during
pain after painting her kitchen. Passive and active his exercise tolerance test. An INITIAL exercise
glenohumeral motions increase pain. The BEST initial prescription that best deals with his problems is walking:
intervention for this acute shoulder condition is: A. five times a week using continuous training, for 60
A. correction of muscle imbalances to allow healing of minutes.
right shoulder supraspinatus tendinitis. B. several times a week using interval training, for 10 to
B. modalities to reduce pain and inflammation as the 15 minute periods.
result of subdeltoid bursitis. C. three times a week using continuous training, for 40
C. rotator cuff strengthening exercises to allow ADL minute sessions.
function following biceps tendinitis. D. three times a week using interval training, for 10 to 15
minutes periods.

401936186.doc
14 Practice Questions

B133 C7 3 B. angina responding quickly to rest and interval rather


133. A valid informed consent for research purposes should than continuous training.
include all of the following elements EXCEPT: C. arrhythmias increasing in frequency, especially atrial
A. a statement ensuring the subject’s commitment to arrhythmias.
participate for the duration of the study. D. delayed cessation of pain following the administration
B. all potential benefits of participation. of nitroglycerin.
C. all reasonable and foreseeable risks and discomforts.
D. an understandable explanation of the purpose and B139 C2 3
procedures to be used. 139. Your patient suffered a left CVA which has left him
hemiparetic on the right side. In his current stage of
B134 C4 1 recovery, he demonstrates strong and dominant hemiplegic
134. During pregnancy, the presence of the hormone relaxin can synergies in his leg. Which activity would NOT be helpful to
lead to abnormal joint hypermobility and pain, MOST break up these synergies?
frequently affecting the: A. assuming the bridging position.
A. carpal and metacarpal joints. B. foot tapping in a sitting position.
B. lumbosacral joints. C. rolling from the hooklying position using lower
C. sacroiliac joints. extremity D1 flexion PNF pattern.
D. spinal facet joints. D. weight shifts in kneeling.

B135 C5 3 B140 C7 3
135. Your patient has severe low back pain as a result of 140. All of the following criteria would designate a patient as
chopping wood three weeks ago. The patient has been homebound and allow for home physical therapy services
receiving ultrasound and strengthening exercises to the low to be approved by Medicare with the EXCEPTION of:
back for two weeks. You have also been applying TENS A. ambulation for short distances causes dyspnea and
and decide to have the patient use TENS at home. As a chest pain.
safety precaution, it is important to instruct the patient to B. dependency on others for all transportation needs.
perform a daily check of the: C. inability to safely leave home unattended.
A. electrodes and electrode jacks. D. leaving the home three times a week to receive
B. electrodes and leads. dialysis.
C. skin and electrodes.
D. skin and leads. B141 C6 1
141. A patient with an above-knee prosthesis is walking by
B136 C6 3 swinging the prosthesis out to the side in an arc during
136. A tilt-in-space wheelchair has been ordered for a patient swing of the amputated limb. Your suspect the prosthesis
with C4 quadriplegia. This type of chair is ordered to: may:
A. facilitate handgrip and propulsion. A. be too long or the knee mechanism too stiff.
B. improve leg position and lower extremity edema. B. have an excessively low lateral wall.
C. improve positioning for pressure relief or for patients C. have an unstable knee unit or is too short.
with extensor spasms. D. have insufficient support from the anterior or posterior
D. improve the patient’s ability to relieve pressure and walls.
transfer independently.
B142 C4 2
B137 C8 2 142. An 87 year-old patient has been hospitalized for three
137. An 82 year-old female has low vision. She recently weeks following a surgical resection of carcinoma of the
returned home from a 2 week hospitalization for colon. He is very weak and you are instructing him in
stabilization of her diabetes. Your goal is to mobilize her ambulation with a walker. He complains of pain in his left
and increase her ambulation level and safety. The BEST shoulder which is aggravated by weight bearing when
intervention strategy for this patient is to: using the walker. You decide to:
A. color code stairs with pastel shades of blue and green A. apply cold (to reduce pain) prior to ambulation
to highlight steps. training.
B. keep window shades wide open to let in as much light B. apply heat in the form of a hot pack.
as possible. C. notify his physician immediately.
C. practice walking by having her look at her feet at all D. observe the shoulder carefully over the next few
times. sessions, documenting any changes in pain.
D. practice walking in areas of high illumination and low
clutter. B143 C5 2
143. A 17 year-old high school football player sprained his left
B138 C3 1 ankle two days ago. He complains of moderate pain (5/10),
138. Your patient has a history of angina pectoris and limited there is moderate swelling which seems to be worsening;
physical activity. As you are supervising him in his second he ambulates with an antalgic gait. In this case, the
exercise class you suspect that his angina is unstable and OPTIMAL interventions to choose are:
may be indicative of a preinfarction state. You determine A. cold whirlpool, followed by elastic compression and
this by the presence of: elevation.
A. angina increasing in intensity and unresponsive to the B. cold/intermittent compression combination followed
nitroglycerin or rest. by elevation.

401936186.doc
Practice Questions 15

C. contrast baths and elastic compression.


D. intermittent compression followed by elevation.

B144 C1 1
144. Your patient diagnosed with left lateral epicondylitis has no
resolution of symptoms after two weeks of treatment. You
begin a re-examination and find the left biceps reflex is 1+.
You should NEXT perform a complete examination of the:
A. cervical spine.
B. shoulder.
C. thoracic spine.
D. wrist.

401936186.doc
16 Practice Questions

B145 C3 2 activities of daily living. An orthopedic surgeon diagnosed


145. An obese patient who is 70 pounds overweight is her problem as adhesive capsulitis. The MOST effective
recovering from a mild myocardial infarction and needs subacute intervention for this patient would include:
cardiovascular conditioning. The exercise class will be A. anterior inferior translatory glides.
used in conjunction with a dietary program to promote B. phonophoresis with dexamethasone cream.
weight reduction. The MOST appropriate exercise C. posterior translatory glides.
prescription for this patient is: D. use of ice packs and a sling.
A. jogging, 10 min. mile (10.2 METs). B151 C8 1
B. swimming, intensity set at 75% age-adjusted heart 151. A 79 year-old patient is referred to physical therapy for an
rate. examination of his functional mobility skills and safety in
C. walking, intensity set at 50% target heart rate. the home environment. The family reports he is
D. walking, intensity set at 85% of heart rate reserve. demonstrating increasing forgetfulness and some memory
deficits. In your examination you would expect to find:
B146 C1 1 A. impairments in short-term memory.
146. A 68 year-old male underwent a total hip replacement B. periods of agitation and wandering, especially in the
(THR) four months ago. He is now referred to physical late afternoon.
therapy for gait evaluation. He demonstrates shortened C. periods of fluctuating confusion.
stride length on the right. The patient MOST LIKELY has: D. significant impairments in long-term memory.
A. lengthened gluteals.
B. shortened hamstrings. B152 C2 1
C. shortened hip flexors. 152. You are examining a patient with vestibular dysfunction.
D. weakened quadriceps. You have the patient sit with the head turned to one side.
You quickly move the patient backward so that the head is
B147 C8 3 extended over the end of the table approximately 30°
147. A child with spastic cerebral palsy is having difficulty below horizontal. The maneuver causes dizziness and
releasing food from her hand into her mouth. Once the vertigo. You report these findings as:
child has brought the food to the mouth it would be helpful A. a negative positional test.
for the caregiver to: B. a positive Clinical Test for Sensory Interaction in
A. give a quick stretch to the finger extensors. Balance (CTSIB).
B. passively extend the fingers. C. a positive Hallpike maneuver.
C. slowly stroke the finger extensors in a distal to D. a positive sharpened Romberg test.
proximal direction.
D. slowly stroke the finger flexors in a distal to proximal B153 C7 3
direction. 153. You are instructing a patient with a lower extremity
amputation in prosthetic gait ambulation. You determine
B148 C2 2 that learning is going well because the patient’s errors are
148. A patient with amyotrophic lateral sclerosis presents with decreasing and overall endurance is increasing. The BEST
early signs of progressive muscle weakness and muscle strategy to promote continued motor learning is to:
cramping. He also has minimal spasticity in his lower A. have the patient begin to walk in the hall outside of
extremities. Your INITIAL exercise prescription should the P.T. clinic.
focus on: B. have the patient continue to practice in the parallel
A. active exercises since resistive exercises are bars until all errors are extinguished.
contraindicated. C. intervene early whenever errors appear before bad
B. low intensity general conditioning exercises. habits become firmly entrenched.
C. maximizing exercise and activity. D. provide continuous feedback after every walking
D. PROM exercise to maintain joint flexibility since active attempt.
exercise is contraindicated.
B154 C3 3
B149 C6 2 154. Which intervention would NOT be appropriate for home
149. In reference to the figure, when lifting a constant load using physical therapy for a patient with cystic fibrosis?
either a stoop lift or squat lift posture, the most significant A. activities to increase endurance.
contributing factor for increasing the lumbar spine B. breathing techniques that will decrease both
compression forces is: respiratory rate and inspiratory volumes.
A. performing the lift with the lumbar spine in a kyphotic C. inspiratory muscle training to increase the strength of
posture using a stoop lift technique. the ventilatory muscles.
B. performing the lift with the lumbar spine in a neutral D. secretion removal techniques to all lobes once or
position rather than a lordotic posture. twice a day.
C. the distance of the load from the base of the spine.
D. the height of the load from the ground. B155 C4 2
155. A 46 year-old woman is referred to your clinic with
B150 C1 2 problems of stress incontinence. She reports loss of control
150. Nearly two months ago, a 50 year-old female noticed that that began with coughing or laughing but now reports she
she had left shoulder pain after walking her dog. This pain has problems even when she exercises (she does aerobics
has progressively worsened. She now is unable to move 3 times a week). The BEST intervention for this patient is:
her left upper extremity overhead while performing A. behavioral modification techniques to reward proper
voiding on schedule.

401936186.doc
Practice Questions 17

B. biofeedback one hour a week to achieve appropriate A. refused to treat this patient.
sphincter control. B. requested more time to read the literature about this
C. functional electrical stimulation three times a week. repair.
D. Kegel exercises several times a day. C. treated the patient as requested.
D. used heat prior to the treatment.
B156 C2 1
156. A 76 year-old patient suffered a cerebral thrombosis four B161 C8 1
days ago. She presents with the following symptoms: 161. Which is NOT considered a normal finding during an
analgesia and thermoanesthesia of the ipsilateral face; examination of a newborn infant?
nystagmus, vertigo, and nausea; dysphagia and dysarthria; A. continuous tremulousness.
ipsilateral Horner’s syndrome; and contralateral loss of B. dramatic skin color changes with change of state.
sensations of pain and temperature of the body. The MOST C. response decrement to repetitive stimuli.
LIKELY site of the lesion is the: D. symmetry in range of motion.
A. anterior inferior cerebellar artery.
B. basilar artery. B162 C6 1
C. internal carotid artery. 162. A patient comes into your clinic with a wheelchair with
D. posterior inferior cerebellar artery. excessive leg length from the seat to the foot plate. You
B157 C5 2 suspect this is resulting in his:
157. An 80 year-old patient with a two inch stage III decubitus A. excessive forward leaning.
ulcer over the left lateral malleolus is referred to you for B. excessive weight bearing on the ischial seat.
electrical stimulation. You note a greenish, pungent C. sacral sitting and sliding forward in the chair.
exudate at the wound site. Your choice of polarity and D. uneven thigh weight distribution.
electrode placement would be: B163 C4 1
A. anode placed in the wound. 163. A patient presents with a large sacral decubitus ulcer that is
B. anode placed proximal to wound. purulent and draining. You need to take a representative
C. cathode placed in the wound. sample of the infected material in order to obtain a
D. cathode placed proximal to wound. laboratory culture. The MOST appropriate method to
culture this wound is to obtain samples from the:
B158 C3 3 A. dressing and exudate in the wound.
158. Running on level gym surface is a suitable Phase 2 B. dressing, exudate, and surrounding bed linen.
outpatient cardiac rehabilitation activity for some patients C. exudate in the wound and the surrounding tissues.
because the MET level is of: D. exudate in the wound.
A. high intensity (15 METs) and should only be used for
late phase II training. B164 C3 3
B. low intensity, (below 5 METs) and little variability 164. A Phase 2 outpatient cardiac rehabilitation program uses
between individuals is expected. circuit training with different exercise stations for the 50
C. moderate intensity, (13 METs) and energy minute program. One station uses arm ergometry. For arm
expenditure will vary significantly among individuals exercise compared to leg exercise at a given workload you
so it should be used with caution. can expect:
D. moderate intensity, (8-9 METS), and can be A. both HR and systolic/diastolic BP will be higher.
successfully alternated with lower intensity walking, (5 B. exercise capacity is reduced due to higher stroke
METs), to produce the right intensity. volumes.
C. HR will be higher while systolic BP will be lower.
B159 C4 2 D. the principal change is higher systolic and diastolic
159. A 27 year-old woman presents with a complete T10 BP.
paraplegia. An extensive neurological work-up has failed to
reveal a specific cause for her paraplegia. Her physician B165 C7 2
has determined that she has a conversion disorder. During 165. A therapist wants to determine whether a treatment was
physical therapy it would be BEST to: effective in reducing lower extremity edema in a group of
A. postpone her rehabilitation until she has had patients with peripheral vascular edema. Volumetric
adequate psychological counseling. measurements are used using water displacement method.
B. treat this patient the same as any other patient with The data was compared to a control group receiving no
paraplegia. treatment. Analysis of this data is BEST done by
C. try to get this patient to recognize the cause of her employing:
malingering. A. ANOVA.
D. use functional electrical stimulation as a means of B. chi square.
demonstrating to her that the muscles function. C. Pearson’s product moment.
D. t-test.
B160 C7 3
160. A physical therapist, a certified cardiopulmonary clinical B166 C8 2
specialist, was asked to treat a patient who had a surgical 166. In planning a physical activity program for an elderly patient
repair of a lacerated index finger flexor tendon. This was with Alzheimer’s disease, it is MOST important that the
the first hand injury that the therapist had ever treated. daily activities:
During the treatment the patient felt a “pop” which was the A. are changed daily to meet the need for variety.
result of a rupture of the newly repaired tendon. The B. are highly structured to reduce anxiety and confusion.
physical therapist in this case should have:

401936186.doc
18 Practice Questions

C. promote involvement and interest through maximum 172. You are currently treating a 32 year-old patient recovering
stimulation. from deep partial thickness burns over the posterior thigh
D. provide maximum opportunities for making choices. and calf which are now healed. Your examination reveals
local tenderness with swelling and pain on movement in
B167 C2 3 the hip area. As you inspect the tissues you detect a
167. Your patient has a complete spinal cord injury at the level palpable mass. Your BEST course of action is to:
of C6. You are instructing his family in exercises to A. cancel therapy for that day to allow tenderness to
maintain his passive range of motion. You want the family subside.
to focus on: B. continue with ROM exercises but proceed gently.
A. keeping all muscles fully ranged through normal C. report these findings promptly to the physician.
ROM. D. use RICE to quiet down the inflammatory response.
B. keeping muscles fully ranged, with hyperflexibility in
the low back extensors and hamstrings. B173 C8 1
C. limiting range of motion in the shoulders to promote 173. A 10 month-old (corrected age) infant born at 23 weeks
stability. gestational age suffered Grade III intraventricular
D. ranging individual muscles according to specific hemorrhage prenatally and was on a ventilator for 2
functional needs. months after birth. During a physical therapy examination,
increased resistance to passive movement is noted in all
B168 C7 3 extremities, but most markedly in the lower extremities. The
168. A physical therapist and physical therapist assistant are infant will probably be diagnosed with:
conducting a cardiac rehabilitation class for twenty A. ataxic cerebral palsy.
patients. The therapist is suddenly called out of the room. B. spastic diplegic cerebral palsy.
The MOST appropriate procedure in this situation is to: C. spastic hemiplegic cerebral palsy.
A. have the patients switch to less intense exercise until D. spastic quadriplegic cerebral palsy.
the therapist returns.
B. have the PTA supervise the class in cool-down B174 C3 1
activities. 174. ECG changes that occur with exercise in an individual with
C. have the PTA supervise the class using the outlined myocardial ischemia and coronary artery disease would be
exercise protocol until the therapist returns. expected to include:
D. terminate the exercises and have the patients monitor A. bradycardia with ST segment depression greater than
their pulses until the therapist returns. 3 mm below baseline.
B. significant arrhythmias early on in exercise with a
B169 C1 3 shortened QRS.
169. The most efficient intervention to regain biceps brachii C. significant tachycardia early on in exercise with ST
strength if the muscle is chronically inflamed and has a segment elevation.
painful arc of motion is: D. tachycardia at a relatively low intensity of exercise
A. biceps curls through partial range of motion with ST segment depression.
B. isokinetic exercises through the full range of motion.
C. isometric exercises at 10 degree intervals through the B175 C1 2
range of motion. 175. A 45 year-old female patient with long term postural
D. isometric exercises at the end range of movement. changes exhibits an excessive forward head and
complains of pain and dizziness when looking upward. The
B170 C5 3 MOST effective physical therapy intervention is:
170. During an ultrasound treatment your patient flinches and A. anterior cervical muscle stretching and postural
states that a strong ache is felt in the treatment area. To reeducation to relieve vertebral artery compression.
accommodate to this patient’s complaint, it would be BEST B. posterior cervical muscle stretching and postural
to: reeducation to relieve vertebral artery compression.
A. add more transmission medium. C. upper cervical Grade III mobilization to reduce
B. decrease the ultrasound frequency. pressure on the cervical spinal nerves.
C. decrease the ultrasound intensity. D. upper cervical joint mobilization Grade II and postural
D. increase the size of the treatment area. reeducation to reduce compression of the cervical
sympathetic ganglia.
B171 C7 3
171. The Back to Work Center, which specializes in work B176 C6 1
conditioning, is scheduled for an accrediting site survey. 176. Your patient has an above-knee prosthesis with a
The appropriate agency to conduct this program is the: prosthetic foot that includes a plantar flexion bumper.
A. Commission on Accreditation of Rehabilitation During ambulation you decide that the bumper is too stiff.
Facilities (CARF). You base this decision on the presence of:
B. Department of Health & Human Services A. foot slap.
C. Joint Commission on Accreditation of Health Care B. high heel rise.
Organizations (JCAHO). C. lateral foot rotation at heel strike.
D. Occupational Safety & Health Administration (OSHA). D. swing phase whip.

B172 C4 2 B177 C3 3

401936186.doc
Practice Questions 19

177. During the course of the physical therapy treatment in the B183 C3 1
ICU, a radial artery line gets pulled (comes out of the 183. A patient is referred for inpatient cardiac rehabilitation. He
artery). The FIRST thing the physical therapist should do is: has a diagnosis of sinus bradycardia. You observe the
A. call for the nurse and elevate the arm above heart patient carefully for worsening signs and symptoms of
level. sinus bradycardia that include:
B. place a brachial blood pressure cuff on the involved A. fibrillatory waves instead of P waves on his ECG.
extremity and inflate the cuff to a level above systole. B. hypotension, chest pain, and dizziness.
C. push the code button in the patient’s room and C. palpitations and racing HR.
elevate the arm above heart level. D. ST segment depression with T wave flattening.
D. re-insert the arterial catheter into the radial artery and B184 C5 3
inflate the pressure pack to a level above systole. 184. Your patient complains of pain in the shoulder region
secondary to subdeltoid bursitis. Pain is reported to be
B178 C2 1 9/10. As part of the plan of care during the acute phase,
178. A 52 year-old patient suffers a CVA resulting in right you elect to use conventional TENS which will modulate
hemisphere damage. This patient will MOST LIKELY the pain primarily by:
exhibit: A. ascending inhibition.
A. hesitancy, requiring more feedback and support. B. descending inhibition.
B. negative, self-deprecating comments and frequent C. gate control mechanisms.
depression. D. stimulation of endorphins.
C. poor judgment with increased safety issues.
D. slow, cautious behaviors. B185 C1 1
185. A thirty-five year-old male laborer describes a sudden
B179 C4 1 onset of back pain while trying to lift a heavy barrel with
179. A 64 year-old patient is referred to your clinic with a another worker. He describes his pain as constant,
diagnosis of herpes zoster with neuralgic pain that has unremitting at an intensity of 10/10 over the last three days.
been intractable for the last 4 months. She has been given He is unable to work, but can drive himself to the clinic for
systemic corticosteroids as part of her medical treatment. It treatment. Medication has not influenced his symptoms. He
is NOT expected that this drug intervention would result in: states he has never had any other back-related symptoms
A. hypokalemia with hypotension and arrhythmias. in the past. MOST LIKELY involved in his symptomatology
B. increased susceptibility to infection. is:
C. muscle cramps, tetany and paresthesias. A. discal dysfunction.
D. osteoporosis and myopathy. B. early degenerative osteoarthritis.
C. neoplastic disease.
B180 C1 1 D. secondary gain.
180. After examining a female patient who was referred to
physical therapy for posterior thoracic pain, you have found B186 C3 1
no musculoskeletal causes for the patient’s symptoms. 186. Following a myocardial infarction, a patient was placed on
Pain may be referred to the thoracic region from the: medications which included a beta-adrenergic blocking
A. appendix. agent. When monitoring this patient’s response to exercise,
B. gall bladder. you expect this drug will cause heart rate to:
C. heart. A. be low at rest and rise linearly as a function of
D. ovary. increasing workload.
B. be low at rest and rise very little with exercise.
B181 C6 3 C. increase proportionally to changes in diastolic blood
181. An 82 year-old patient with a transfemoral amputation is to pressure.
be fitted with a temporary prosthesis containing a SACH D. increase proportionally to changes in systolic blood
prosthetic foot. This prosthetic foot: pressure.
A. absorbs energy through a series of bumpers,
permitting sagittal plane motion only. B187 C7 3
B. allows full sagittal and frontal plane motion. 187. You are working in an outpatient clinic that has provider
C. allows limited sagittal plane motion with a small contracts with many types of insurance plans. A large
amount of mediolateral motion. percentage of the clinic’s patients have insurance coverage
D. is an articulated foot with multiplanar motion. that has a contract that pays a flat rate of reimbursement
per episode of care. Your supervisor tells you never to treat
B182 C8 2 these patients for more than five visits. You should:
182. An 82 year-old patient demonstrates significant A. ignore your supervisor and treat the patients at the
proprioceptive losses in both lower extremities, distal frequency and duration appropriate for their condition.
greater than proximal. Effective compensatory strategies to B. keep track of all your patients insured by this plan,
assist in ambulation training would consist of: and make certain they average less than or equal to
A. constant verbal cuing concerning his foot position. five visits in aggregate.
B. having him watch the position of his feet while C. resign from the clinic and report the supervisor to the
standing. state’s licensing board.
C. limiting his ambulation and focusing on wheelchair D. speak to your supervisor about concerns you have
mobility. that the policy is in conflict with the Code of Ethics.
D. walking on smooth tile floors.
B188 C4 3

401936186.doc
20 Practice Questions

188. Your patient is a 65 year-old with a 20 year history of B. low scores on the Geriatric Depression Scale.
diabetes. Changes include vascular insufficiency and C. sleep apnea
diminished sensation of both feet with poor healing of a D. withdrawal, symptoms of fatigue and weight loss.
superficial skin lesion. It is important that he understand the
precautions and guidelines on foot care for people with B193 C7 3
diabetes. All of the following will BEST accomplish this goal 193. A 40 year-old ironworker has been laid off his construction
except instructing the patient to: job for the last two years. He has a wife who is a
A. inspect the skin daily for inflammation, swelling, homemaker and seven small children. They are currently
redness, blisters, or wounds. on welfare. While raking leaves he has a stroke and is
B. use daily hot soaks of feet followed by an application admitted to the hospital. In this case, the third party payer
of petroleum jelly to moisturize skin. that would provide assistance is:
C. wash the feet daily with mild soap, dry thoroughly, A. Medicaid.
and hydrate with moisturizing lotion. B. Medicare Part A.
D. wear oxford-type or jogging shoes that allow C. Social Security.
adequate room and change shoes frequently. D. Worker’s Compensation

B189 C1 1 B194 C3 1
189. A patient complained of an inability to fully extend his knee 194. A patient has been on bedrest for two days following
when running. During your examination you determine that revascularization surgery involving triple coronary artery
he cannot fully extend his knee while positioned in supine bypass graft. During his therapy session you suspect he
with the foot dorsiflexed and the hip flexed first to 60 may be developing complications of deep vein
degrees and then 90 degrees. The tightness is most likely thrombophlebitis. The MOST important signs of this
caused by the: potential medical emergency are:
A. gastrocnemius. A. dyspnea and tachypnea with leg pain.
B. hamstrings. B. pain in the calf after exercising, and diaphoresis.
C. psoas. C. paresthesias and a cyanotic cold extremity.
D. soleus. D. tenderness, aching, and swelling in the calf.
B190 C1 2
190. A nineteen year-old male soccer player sustained a Grade B195 C1 1
II inversion ankle sprain two weeks ago. Interventions in 195. A patient referred to you for TMJ dysfunction states that
the early subacute phase of rehabilitation should include: she has had three episodes of her jaw locking in an open
A. closed-chain lower extremity strengthening, position. The MOST LIKELY cause of her problems is the:
proprioceptive exercises, and an orthosis. A. disc.
B. lower extremity strengthening for soccer and other B. lateral pterygoid muscle.
ADL-related tasks. C. masseter muscle
C. open-chain lower extremity exercises, crutch training, D. retrodiscal lamina.
and ankle wrap. B196 C4 2
D. rest, ice, compression and elevation (RICE). 196. A 62 year-old patient is referred to physical therapy after a
fall and ORIF for a fracture of her right wrist. During your
B191 C5 2 initial examination, you observe that her skin and eyes
191. A 46 year-old patient was referred to you following a have a yellowish hue. When you question her about this
tendon transfer of the extensor carpi radialis longus. The she tells you that she noticed it herself starting about 3 or 4
muscle strength tested 2/5 in spite of previous intensive days ago. Your BEST course of action is to:
therapy. You choose biofeedback to assist in progressively A. document your findings and consult with her
increasing active motor recruitment. Initially, the EMG physician by phone after the treatment session.
biofeedback protocol should consist of: B. document your findings and continue on with her
A. high detection sensitivity with recording electrodes treatment.
placed closely together. C. send a written copy of your examination, including
B. high detection sensitivity with recording electrodes your findings of her integumentary condition to her
placed far apart. referring physician.
C. low detection sensitivity with recording electrodes D. write up your examination, then treat her problem with
placed closely together. whirlpool and massage.
D. low detection sensitivity with recording electrodes
placed far apart. B197 C1 2
197. A 75 year-old inpatient received a cemented total hip
B192 C8 1 replacement two days ago. The physical therapy plan of
192. You receive a referral from the nurse case manager for care should have as its number one priority:
physical therapy evaluation for an ambulation program. An A. active range of motion exercises and early
82 year-old has lost functional independence lately after ambulation using a walker, nonweightbearing.
the recent death of his wife. His past medical history B. passive range of motion exercises and gait training
includes stroke with minimal residual disability. Currently he using crutches, weightbearing to tolerance.
no longer goes out of his house and rarely even gets out of C. patient education regarding positions and movements
his chair anymore. During the initial session you determine to avoid.
that depression is likely the cause of his increasing D. use of a CPM machine followed by gait training in the
inactivity based on the presence of: parallel bars.
A. complaints of increasing dizziness and palpitations

401936186.doc
Practice Questions 21

B198 C7 3
198. A patient is referred to you by an orthopedist with the
diagnosis of impingement syndrome of the shoulder. The
initial examination reveals signs and symptoms that are not
consistent with this diagnosis and more consistent with
thoracic spine pain and dysfunction. You treat the patient
consistent with your findings without communicating with
the referring physician. Months later you are sued by the
patient’s estate for failure to order a chest X-ray. The
patient died of undiagnosed metastatic lung cancer. You
are:
A. not legally licensed to order a chest X-ray, therefore
cannot be held responsible for the patient’s death
from metastatic lung cancer.
B. not responsible for the incorrect diagnosis because
your treatment was appropriate for the findings
identified on the initial examination.
C. responsible for communicating findings to the
referring physician when the findings are inconsistent
with a referring physician’s diagnosis.
D. responsible for making this diagnosis of possible
cancer consistent with your examination of this
patient.

B199 C5 3
199. A 44 year-old patient with lymphedema is receiving
intermittent compression to the left lower extremity. An
appropriate treatment time for home administration would
be up to:
A. 2 hours/3 times per week.
B. 30 minutes/daily.
C. 8 hours/3 times per week.
D. 8 hours/daily.

B200 C6 2
200. A patient with a 10 year history of multiple sclerosis
demonstrates 3+ extensor tone in both lower extremities.
You need to order her a wheelchair. It would be BEST to
recommend a:
A. electric wheelchair with toe loops.
B. standard wheelchair with a 30 degree reclining back.
C. standard wheelchair with elevating leg rests.
D. tilt-in-space wheelchair with a pelvic belt.

401936186.doc
22 Practice Questions

1. reaching for a multicolored object while in an unsupported, 44. within a few days following surgery to preserve tendon
guarded sitting position. gliding.
2. change to grade II mobilizations to reduce the joint and soft 45. decreased pain and tenderness at the fracture sites.
tissue irritation. 46. optic chiasm.
3. use frequent reinforcements for all desired behaviors. 47. common peroneal nerve.
4. isokinetic training using low resistance at fast movement 48. accuracy of the measurements increases at higher
speeds. saturation levels.
5. halo orthosis. 49. anterior cruciate ligament.
6. administer supplemental O2. 50. pendulum exercises.
7. electrical silence. 51. intense cold, burning, and aching, followed by numbness.
8. passive ROM exercise. 52. individual facilities modifying its format and implementation
9. stretching of scalenes and sternocleidomastoid muscles for schedule.
early thoracic outlet syndrome symptoms. 53. practice isolated small range quadriceps eccentric control
10. hypoglycemia; you immediately administer orange juice work in standing and continue with the straight cane.
and notify his physician. 54. progressive resistance training for the quadriceps.
11. modify her exercise prescription to decrease the intensity 55. the patient’s employer when the condition is work-related.
and duration but maintain a frequency of 3 times/week. 56. combining the usage of a lumbar support, armrests, and
12. provide real-life examples that link learning to her personal increase the angle between the seat pan and backrest to
experiences. between 90-110 degrees.
13. daily, following the initial examination. 57. suprascapular nerve.
14. exercise-induced bronchospasm. 58. needs assessment performed by a physical therapist.
15. isotonic exercises using ankle weights. 59. repetition of movements and positions that provoke
16. strengthen hip extensors through bridging. dizziness and vertigo.
17. Sulfamylon 60. inadequate knee friction or too little tension in the extension
18. cataracts aid.
19. an environmental and daily structure in which the patient is 61. give him some water to drink and report your findings to his
best able to process stimuli cognitively. physician immediately.
20. sitting, weight bearing on extended left arm, weight shifting. 62. greater than 1 mm ST segment depression, horizontal or
21. the spread of scores with cold treatment demonstrates downsloping.
variability is greater. 63. neutral warmth.
22. supraspinatus 64. ipsilateral weakness and loss of position sense and
23. restrict all movement. vibration below the lesion level with contralateral loss of
24. weak deltoids. pain & temperature sensation.
25. the infant’s head is turned to the right. 65. demonstrate the task, then practice with the patient.
26. provide a questionnaire to all participants two weeks before 66. home health agency rehabilitation services.
the scheduled session. 67. socket is aligned too far forward or tilted anteriorly.
27. maximum use of demonstration and gesture. 68. sustained maximal inspiration training with an incentive
28. provide suggestions to nursing for positioning for optimal spirometer.
motor development. 69. a high back.
29. closed-chain functional strengthening of the quadriceps 70. phonophoresis to reduce inflammation of the tendinous
femoris and hamstrings emphasizing regaining terminal sheaths.
knee extension. 71. a pommel to keep hips abducted.
30. a black and white face, with a red nose, held approximately 72. sensory losses and sensory organization of balance.
nine inches from the infant’s eyes, moved horizontally. 73. the PT who is solely responsible for assessing the
31. position in sidelying, check to see if the airway is open, and competence of the PTA under their supervision.
immediately call for emergency assistance. 74. complete the written component of the discharge summary
32. dependent variable. at another time and evaluate this new patient.
33. slowed respiratory rate. 75. ankle plantarflexion and anterior pelvic tilt.
34. posterior plastic shell. 76. L4 nerve root compression.
35. Stage 1 ulcer formation. 77. take his BP pre and post exercise, using his non-shunt
36. four electrodes with current flow diagonal to the spinal arm.
column. 78. three-point.
37. 9-10 months. 79. since a no treatment group was not used for comparison,
38. postural reeducation, soft tissue mobilization, and dynamic no conclusions can be reached.
stabilization. 80. stop the exercise and notify the physician immediately.
39. ask about previous work and other experiences related to 81. visual agnosia.
transferring individuals. 82. drugs that produce essential tremor.
40. skilled nursing facility. 83. examination for an orthosis.
41. taste sensation over the anterior tongue as well as motor 84. grasp the handrims posteriorly and pull them forward
function of facial muscles. abruptly and forcefully.
42. decreased trunk rotation with shorter steps. 85. amyotrophic lateral sclerosis with dysphagia and
43. orthostatic hypotension and dizziness. diminished gag reflex.
86. partial squats progressing to lunges.

401936186.doc
Practice Questions 23

87. low lumbar (L4-L5). 127. modalities to reduce pain and inflammation as the result of
88. paraffin with the patient positioned in a recumbent position. subdeltoid bursitis.
89. front wheel rolling walker that folds. 128. alterations in memory, confusion, and disorientation.
90. forward, producing posterior tilting of the pelvis. 129. the applicant’s marital status.
91. demonstrate PROM techniques, offering corrections as 130. cause femoral anteversion and knee strain.
they perform the exercises for you. 131. dorsal column/lemniscal pathways or somatosensory
92. facial nerve pathology: Bell’s palsy. cortex.
93. standard postural drainage for the lateral basal segment, 132. several times a week using interval training, for 10 to 15
left lower lobe. minute periods.
94. these cognitive symptoms are intermittent and come on 133. a statement ensuring the subject’s commitment to
suddenly, often at night. participate for the duration of the study.
95. analysis of variance. 134. sacroiliac joints.
96. utilize demonstration and gesture to get the idea across of 135. skin and electrodes.
what you want him to do. 136. improve positioning for pressure relief or for patients with
97. insufficient knee friction. extensor spasms.
98. plastic cervical orthosis and axillary splints (airplane 137. practice walking in areas of high illumination and low
position). clutter.
99. compression of the common peroneal nerve. 138. angina increasing in intensity and unresponsive to the
100. bridging with arms at sides. nitroglycerin or rest.
101. give the patient’s family Medicare notification of 139. foot tapping in a sitting position.
noncoverage information, and carefully explain it, and their 140. dependency on others for all transportation needs.
options, which could include paying for the care out-of- 141. be too long or the knee mechanism too stiff.
pocket. 142. notify his physician immediately.
102. call the patient’s physician due to signs of theophylline 143. cold/intermittent compression combination followed by
toxicity. elevation.
103. there may be a dislocation at C1/C2 due to ligamentous 144. cervical spine.
laxity. 145. walking, intensity set at 50% target heart rate.
104. passive range of motion while emphasizing 146. shortened hip flexors.
scapulohumeral rhythm. 147. slowly stroke the finger extensors in a distal to proximal
105. weakness or severe spasticity of the quadriceps. direction.
106. povidone-iodine. 148. low intensity general conditioning exercises.
107. leg ergometry is highly correlated with RPE while arm 149. the distance of the load from the base of the spine.
ergometry is only moderately correlated. 150. anterior inferior translatory glides.
108. immediately return the patient to the referring physician 151. impairments in short-term memory.
with documentation indicating that treatment was 152. a positive Hallpike maneuver.
ineffective. 153. have the patient begin to walk in the hall outside of the P.T.
109. continuous US @ 3 MHz. clinic.
110. facilitation of early movement in synergistic patterns 154. breathing techniques that will decrease both respiratory
followed quickly by movement patterns out-of-synergy. rate and inspiratory volumes.
111. bilateral KAFO’s. 155. Kegel exercises several times a day.
112. strengthen the hip abductors on the right side. 156. posterior inferior cerebellar artery.
113. decreased loading of the knee by the quadriceps femoris 157. cathode placed in the wound.
muscle. 158. moderate intensity, (8-9 METS), and can be successfully
114. calcium alginate dressings. alternated with lower intensity walking, (5 METs), to
115. centralized gnawing pain with loss of postural control produce the right intensity.
during lifting activities. 159. treat this patient the same as any other patient with
116. involving the patient in decision making, emphasizing paraplegia.
safety and independent performance. 160. refused to treat this patient.
117. use a toe wedge. 161. continuous tremulousness.
118. high intensity workloads for short durations. 162. sacral sitting and sliding forward in the chair.
119. increased HR and RR along with dilated pupils. 163. exudate in the wound.
120. ambulation for exercise with a reciprocating gait orthosis 164. both HR and systolic/diastolic BP will be higher.
and Lofstrand crutches. 165. t-test.
121. placebo effect. 166. are highly structured to reduce anxiety and confusion.
122. respiratory alkalosis. 167. ranging individual muscles according to specific functional
123. avoid exercise during periods of peak insulin activity. needs.
124. will likely be resistant to activity training if unfamiliar 168. have the PTA supervise the class using the outlined
activities are used. exercise protocol until the therapist returns.
125. focus on knowledge of results and visual inputs. 169. isometric exercises at 10 degree intervals through the
126. pad over the transverse arch. range of motion.
170. decrease the ultrasound intensity.
171. Commission on Accreditation of Rehabilitation Facilities 174. tachycardia at a relatively low intensity of exercise with ST
(CARF). segment depression.
172. report these findings promptly to the physician. 175. posterior cervical muscle stretching and postural
173. spastic diplegic cerebral palsy. reeducation to relieve vertebral artery compression.
176. lateral foot rotation at heel strike.

401936186.doc
24 Practice Questions

177. place a brachial blood pressure cuff on the involved


extremity and inflate the cuff to a level above systole.
178. poor judgment with increased safety issues.
179. muscle cramps, tetany and paresthesias.
180. gall bladder.
181. allows limited sagittal plane motion with a small amount of
mediolateral motion.
182. having him watch the position of his feet while standing.
183. hypotension, chest pain, and dizziness.
184. gate control mechanisms.
185. secondary gain.
186. be low at rest and rise very little with exercise.
187. speak to your supervisor about concerns you have that the
policy is in conflict with the Code of Ethics.
188. use daily hot soaks of feet followed by an application of
petroleum jelly to moisturize skin.
189. gastrocnemius.
190. closed-chain lower extremity strengthening, proprioceptive
exercises, and an orthosis.
191. high detection sensitivity with recording electrodes placed
closely together.
192. withdrawal, symptoms of fatigue and weight loss.
193. Medicaid.
194. tenderness, aching, and swelling in the calf.
195. disc.
196. document your findings and consult with her physician by
phone after the treatment session.
197. patient education regarding positions and movements to
avoid.
198. responsible for communicating findings to the referring
physician when the findings are inconsistent with a
referring physician’s diagnosis.
199. 8 hours/daily.
200. tilt-in-space wheelchair with a pelvic belt.

401936186.doc

Вам также может понравиться